No tags match your search
Mcircumcircle
geometry
algebra
number theory
trigonometry
inequalities
function
polynomial
probability
combinatorics
calculus
analytic geometry
3D geometry
quadratics
AMC
ratio
AIME
modular arithmetic
logarithms
LaTeX
complex numbers
rectangle
conics
circumcircle
geometric transformation
induction
integration
floor function
system of equations
counting
perimeter
rotation
trig identities
vector
trapezoid
search
graphing lines
angle bisector
prime numbers
slope
parallelogram
AMC 10
symmetry
relatively prime
parabola
Diophantine equation
Vieta
angles
factorial
Inequality
domain
No tags match your search
MG
Topic
First Poster
Last Poster
Polynomial
Z_. 1
N
an hour ago
by rchokler
Let
be an integer greater than zero. Then, the value of the sum of the reciprocals of the cubes of the roots of the equation
is equal to:

![\[
mx^4 + 8x^3 - 139x^2 - 18x + 9 = 0
\]](http://latex.artofproblemsolving.com/a/3/a/a3a9bca412e927632e4921c51f2a6593409031d9.png)
1 reply
Existence of perfect squares
egxa 2
N
an hour ago
by pavel kozlov
Source: All Russian 2025 10.3
Find all natural numbers
for which there exists an even natural number
such that the number
is a perfect square.


![\[
(a - 1)(a^2 - 1)\cdots(a^n - 1)
\]](http://latex.artofproblemsolving.com/9/c/a/9caf4eeb82ff46b5ba55ab4b6bc28f0cace586ec.png)
2 replies

IMO 2014 Problem 4
ipaper 169
N
2 hours ago
by YaoAOPS
Let
and
be on segment
of an acute triangle
such that
and
. Let
and
be the points on
and
, respectively, such that
is the midpoint of
and
is the midpoint of
. Prove that the intersection of
and
is on the circumference of triangle
.
Proposed by Giorgi Arabidze, Georgia.

















Proposed by Giorgi Arabidze, Georgia.
169 replies
Inequalities
Scientist10 1
N
3 hours ago
by Bergo1305
If
, then prove that the following inequality holds:

![\[
\sum_{\text{cyc}} \sqrt{1 + \left(x\sqrt{1 + y^2} + y\sqrt{1 + x^2}\right)^2} \geq \sum_{\text{cyc}} xy + 2\sum_{\text{cyc}} x
\]](http://latex.artofproblemsolving.com/9/5/5/955863a0cf8747ae45b736b0631f243d3908eb84.png)
1 reply
Tangents forms triangle with two times less area
NO_SQUARES 1
N
3 hours ago
by Luis González
Source: Kvant 2025 no. 2 M2831
Let
be triangle, inscribed in parabola. Tangents in points
forms triangle
. Prove that
. (
is area of triangle
).
From F.S.Macaulay's book «Geometrical Conics», suggested by M. Panov






From F.S.Macaulay's book «Geometrical Conics», suggested by M. Panov
1 reply
FE solution too simple?
Yiyj1 9
N
3 hours ago
by jasperE3
Source: 101 Algebra Problems from the AMSP
Find all functions
such that the equality
holds for all pairs of real numbers
.
My solution
I feel like my solution is too simple. Is there something I did wrong or something I missed?



My solution
Clearly,
is an obvious solution. Now, let
. Then, we have
or
. Therefore, the solutions are
.





I feel like my solution is too simple. Is there something I did wrong or something I missed?
9 replies
interesting function equation (fe) in IR
skellyrah 2
N
3 hours ago
by jasperE3
Source: mine
find all function F: IR->IR such that

2 replies
Complicated FE
XAN4 1
N
3 hours ago
by jasperE3
Source: own
Find all solutions for the functional equation
, in which
: 
Note: the solution is actually quite obvious -
, but the proof is important.
Note 2: it is likely that the result can be generalized into a more advanced questions, potentially involving more bash.



Note: the solution is actually quite obvious -

Note 2: it is likely that the result can be generalized into a more advanced questions, potentially involving more bash.
1 reply
Find all sequences satisfying two conditions
orl 34
N
3 hours ago
by YaoAOPS
Source: IMO Shortlist 2007, C1, AIMO 2008, TST 1, P1
Let
be an integer. Find all sequences
satisfying the following conditions:
![\[ \text{ (a) } a_i \in \left\{0,1\right\} \text{ for all } 1 \leq i \leq n^2 + n;
\]](//latex.artofproblemsolving.com/3/c/5/3c509ec2e9013e8d3be492c8eb44a7c33841b74e.png)
![\[ \text{ (b) } a_{i + 1} + a_{i + 2} + \ldots + a_{i + n} < a_{i + n + 1} + a_{i + n + 2} + \ldots + a_{i + 2n} \text{ for all } 0 \leq i \leq n^2 - n.
\]](//latex.artofproblemsolving.com/9/7/d/97d2a467d1c0dc8594ec024c3bb9b8c87ee85b19.png)
Author: Dusan Dukic, Serbia


![\[ \text{ (a) } a_i \in \left\{0,1\right\} \text{ for all } 1 \leq i \leq n^2 + n;
\]](http://latex.artofproblemsolving.com/3/c/5/3c509ec2e9013e8d3be492c8eb44a7c33841b74e.png)
![\[ \text{ (b) } a_{i + 1} + a_{i + 2} + \ldots + a_{i + n} < a_{i + n + 1} + a_{i + n + 2} + \ldots + a_{i + 2n} \text{ for all } 0 \leq i \leq n^2 - n.
\]](http://latex.artofproblemsolving.com/9/7/d/97d2a467d1c0dc8594ec024c3bb9b8c87ee85b19.png)
Author: Dusan Dukic, Serbia
34 replies

IMO Shortlist 2011, G4
WakeUp 125
N
3 hours ago
by Davdav1232
Source: IMO Shortlist 2011, G4
Let
be an acute triangle with circumcircle
. Let
be the midpoint of
and let
be the midpoint of
. Let
be the foot of the altitude from
and let
be the centroid of the triangle
. Let
be a circle through
and
that is tangent to the circle
at a point
. Prove that the points
and
are collinear.
Proposed by Ismail Isaev and Mikhail Isaev, Russia

















Proposed by Ismail Isaev and Mikhail Isaev, Russia
125 replies
